Power Tower with

แชร์
ฝัง
  • เผยแพร่เมื่อ 27 มิ.ย. 2024
  • Grant Sanderson of 3Blue1Brown brings a power tower puzzle to Oxford for us to solve! Featuring self-similarity, convergence versus divergence and cobweb maps, the solution is not quite as straightforward as it first seems...
    Find Grant's amazing channel 3Blue1Brown here: / @3blue1brown
    Produced by Dr Tom Crawford at the University of Oxford.
    For more maths content check out Tom's website tomrocksmaths.com/
    You can also follow Tom on Facebook, Twitter and Instagram @tomrocksmaths.
    / tomrocksmaths
    / tomrocksmaths
    / tomrocksmaths
    Get your Tom Rocks Maths merchandise here:
    beautifulequations.net/collec...
    Tom Rocks Maths logo by Nathanial Butt: nat_von_b?...

ความคิดเห็น • 163

  • @TomRocksMaths
    @TomRocksMaths  3 ปีที่แล้ว +55

    If you missed the first video in the series, you can watch me and grant take part in 'Maths Speed Dating' here: th-cam.com/video/Xh2LVy7B5q0/w-d-xo.html

  • @KramerEspinoza
    @KramerEspinoza ปีที่แล้ว +52

    I love the way you guys presented this, showing that math is a human endeavor, that even the brightest among us can be momentary “fooled” and be humble and enthusiastic about it.

  • @admiralhyperspace0015
    @admiralhyperspace0015 3 ปีที่แล้ว +115

    I just don't know. Whenever I see grant I am filled with this calm feeling. I really like his personality and vibe. I wish I was that gentle and soothing. I really do.

  • @devenhull3677
    @devenhull3677 3 ปีที่แล้ว +342

    Two of the most attractive men in math

    • @hannahsrzich5110
      @hannahsrzich5110 3 ปีที่แล้ว +2

      Bars down here bruv

    • @ImaginaryMdA
      @ImaginaryMdA 3 ปีที่แล้ว +4

      Tom and the camera man? jkjk

    • @bryanfuentes1452
      @bryanfuentes1452 3 ปีที่แล้ว +1

      love this joke

    • @arlenestanton9955
      @arlenestanton9955 3 ปีที่แล้ว +1

      Tom is a freak

    • @webgpu
      @webgpu 2 ปีที่แล้ว

      @@arlenestanton9955 you mean his piercings and tattoos?

  • @NP-zl7dz
    @NP-zl7dz 3 ปีที่แล้ว +102

    Damn, Grant's wearing a wedding ring, I'm too late.

    • @farazriyaz9078
      @farazriyaz9078 3 ปีที่แล้ว +9

      Polygamy is a thing. Now go for it!

    • @webgpu
      @webgpu 2 ปีที่แล้ว

      Grant's a Conservative. That's the proper way of constituting a family. (the freaks that like to behave out of Judeo-Christian ethics could all gather in their own country and disintegrate there by themselves)

  • @garygallagher7341
    @garygallagher7341 ปีที่แล้ว +15

    This is beautiful - the humility and collegiality on display here is inspiring.

  • @emilianpopa1402
    @emilianpopa1402 5 หลายเดือนก่อน +1

    13:45 "What would it mean actually?" 😅 The way he said it made me smile.

  • @varunraju1569
    @varunraju1569 3 ปีที่แล้ว +113

    This is so good! Love the collab!

    • @TomRocksMaths
      @TomRocksMaths  3 ปีที่แล้ว +8

      Thanks Varun - glad you enjoyed it!

  • @noonesperfect
    @noonesperfect 3 ปีที่แล้ว +35

    Mr. Grant also covered this topic in one of his lock-down series, good to see Mr. Tom collabed on same topic, it refreshes a lot things. Surely one of the rare topic we can get over internet so precisely :)

  • @MohitKumar-ge9hw
    @MohitKumar-ge9hw 3 ปีที่แล้ว +12

    I love how this puzzle links to iterated maps and dynamical systems! At first the connection isn't obvious but then Grant brings in the idea of seeing it as a function being repeated. This allows one to bring in the machinery of iterated maps and dynamical systems. As a novice of the said topics, it was nice to realise that one of the solutions is stable and the other is unstable and they coalesce in the form of a saddle node bifurcation at the value found in the video.

    • @TomRocksMaths
      @TomRocksMaths  3 ปีที่แล้ว +2

      Grant picked a great puzzle :)

  • @jocantu
    @jocantu 3 ปีที่แล้ว +67

    Towers in two math channels at the same time? This is a conspiracy. (⌐■_■)

    • @TomRocksMaths
      @TomRocksMaths  3 ปีที่แล้ว +6

      This I did not know, which other channel??

    • @dekunut6416
      @dekunut6416 3 ปีที่แล้ว +6

      @@TomRocksMaths numberphile :)

    • @TomRocksMaths
      @TomRocksMaths  3 ปีที่แล้ว +10

      Clearly we are both right on topic without even realising it...

    • @shk439
      @shk439 3 ปีที่แล้ว +1

      @@TomRocksMaths Also I think blackpenredpen

    • @bachirblackers7299
      @bachirblackers7299 3 ปีที่แล้ว

      Blackpen. Redpen made it in a different way with the limits of convergence for both x n y values . More over still there is more to tell in this videos at least showing the case where a to the power of x crosses the y=x twice when we would be facing that bounded area between the curve n the y=x . Can we even think of integration !!!

  • @paradoxica424
    @paradoxica424 3 ปีที่แล้ว +32

    hi tom. the lower bound of the base is exp(-e), a proof can be found in a mock exam that i co-authored. BoS Trials, 2019 Mathematics Extension 1, Q13 (a)

  • @bonzaiii3
    @bonzaiii3 3 ปีที่แล้ว +3

    Two such great persons in maths and charismatic, hope to see more vids of these guys together.

  • @em8136
    @em8136 3 ปีที่แล้ว +13

    I dont know how i got here but I'm not disappointed

    • @TomRocksMaths
      @TomRocksMaths  3 ปีที่แล้ว +8

      the YT algorithm works in mysterious ways

  • @AEmiliusLives
    @AEmiliusLives 3 ปีที่แล้ว +8

    Love these collabs with Grant :D

  • @motivationnation999
    @motivationnation999 ปีที่แล้ว +2

    This is so above my level of knowledge and I thoroughly enjoyed watching you both talk through it.

  • @LeonKayombo-yk4tc
    @LeonKayombo-yk4tc 9 หลายเดือนก่อน +1

    I like Dr Tom's glittering laughter.... Always...showing humility in mathematics! Ganbate kudasai !

  • @esb1874
    @esb1874 3 ปีที่แล้ว +19

    "To solve an equation, you have to know that the equation has a solution" would be fitting for both videos seeing as it is precisely one of open problems of Navier-Stokes equations!

  • @_1derscore
    @_1derscore 7 วันที่ผ่านมา

    if maths split into two personalities

  • @ShromannMajumder
    @ShromannMajumder 2 ปีที่แล้ว +4

    I remember doing this question in high school, and the way I tackled it was.
    x^x^x… = 2
    => log2(x^x…) = log2(2)
    => x^x^x… * log2(x) = 1
    => 2 * log2(x) = 1
    => log2(x) = 1/2
    => x = 2^(1/2)
    That’s how I got sqrt(2) haha

  • @Holobrine
    @Holobrine 3 ปีที่แล้ว +6

    9:46 That would appear to intersect at x=4, actually. But if you tried to approach it, you can’t because it’s like an unstable equilibrium. The fact remains that sqrt(2)^4 = 4.

    • @TomRocksMaths
      @TomRocksMaths  3 ปีที่แล้ว +4

      It's true that it is an intersection point, but the iterative procedure that Grant defines will never end up there (unless you start at exactly that point).

    • @Holobrine
      @Holobrine 3 ปีที่แล้ว +2

      Tom Rocks Maths Yeah, that’s what I meant by unstable equilibrium.

  • @ishaanjain4211
    @ishaanjain4211 3 ปีที่แล้ว +24

    Math men math men
    Math math math men men
    With Tom Crawford and/or Grant Sanderson

    • @TomRocksMaths
      @TomRocksMaths  3 ปีที่แล้ว +3

      LOVE this. What's the tune?

    • @ishaanjain4211
      @ishaanjain4211 3 ปีที่แล้ว +4

      Check out Jay Foreman's series 'Map Men'

    • @asheep7797
      @asheep7797 3 หลายเดือนก่อน

      men

    • @ishaanjain4211
      @ishaanjain4211 3 หลายเดือนก่อน

      @@asheep7797 of course how could I have missed that

  • @sumchief
    @sumchief 5 หลายเดือนก่อน

    this is awesome

  • @olivierbeaudry-ogden938
    @olivierbeaudry-ogden938 2 หลายเดือนก่อน

    I love the fact that 2 and 4 is possible if you look at the graoh depending where u start u can get 2 or 4

  • @davidherrera8432
    @davidherrera8432 3 ปีที่แล้ว +1

    This is beautiful, what a great video!

  • @samayahone3497
    @samayahone3497 2 ปีที่แล้ว

    this is so far above my level and yet i thoroughly enjoyed the whole thing, hopefull one day i will be able to see this and understand what all your maths means lmao

  • @R3DD10N
    @R3DD10N 3 ปีที่แล้ว +5

    You are making maths cool

    • @TomRocksMaths
      @TomRocksMaths  3 ปีที่แล้ว +1

      Thanks Eric!!

    • @R3DD10N
      @R3DD10N 3 ปีที่แล้ว

      @@TomRocksMaths could I by any chance talk to you about the Navier-Stokes equations, I have some ideas about them and would like to bring them to an expert. My email is ejhoman92@gmail.com

  • @axelperezmachado3500
    @axelperezmachado3500 3 ปีที่แล้ว +4

    Great vid! Another way to quickly find the range of convergence is to plot the y=x^y graph. That's using the fact that x^x^... = y. You will find that the only x's that satisfy that equation are in (0 ; e^(1/e)]

    • @TomRocksMaths
      @TomRocksMaths  3 ปีที่แล้ว +2

      Nice!

    • @aprendiendoC
      @aprendiendoC ปีที่แล้ว +1

      I tried plotting y=x^y in Wolfram Alpha but I don't know where you get the result that the only x's that satisfy that equation are in (0 ; e^(1/e)]

    • @axelperezmachado3500
      @axelperezmachado3500 ปีที่แล้ว +1

      @@aprendiendoC By graphing y=x^y you get a curve that is equivalent to x=y^x (which defines a real function f). We are looking for a subdomain in which this function is inyective beacuse x^x^...= y defined as a limit must be unique (a limit can`t have multiple values). It turns out that that subdomain is (0; argmax(f)] where argmax(f) turns out to be e^(1/e) by just using standard calculus tools.

  • @tsvibenschar4135
    @tsvibenschar4135 2 ปีที่แล้ว

    the domain max of x^y=y is the range max of y^x=x, which is point e,e^(1/e) by the first and second derivative tests (they get messy). So the domain max of the original x^y=y is e^(1/e)~1.44466, which maps to e.

  • @christianorlandosilvaforer3451
    @christianorlandosilvaforer3451 ปีที่แล้ว +1

    absolutly dope collabo

  • @marshmelows
    @marshmelows 3 ปีที่แล้ว +4

    "The whole property of infinity is that, you add 1 and you don't change size" Wow, so simple

    • @letspass3465
      @letspass3465 3 ปีที่แล้ว +1

      *add

    • @TIO540S1
      @TIO540S1 3 ปีที่แล้ว +2

      I learned that an infinite set is defined as "any set for which there exists a one to one correspondence with a proper subset of that set."

    • @marshmelows
      @marshmelows 3 ปีที่แล้ว +1

      @@letspass3465 thanks!

    • @melvinnjoroge5497
      @melvinnjoroge5497 3 ปีที่แล้ว

      @@TIO540S1 cool!

  • @extremestudy101
    @extremestudy101 ปีที่แล้ว

    This should be an Oxford Interview Question for students trying to apply for maths

  • @Zoxesyr
    @Zoxesyr 3 ปีที่แล้ว +11

    Grant seems a lot less uncomfortable around Tom this time.

  • @Someone-cr8cj
    @Someone-cr8cj 3 ปีที่แล้ว +6

    i think i watched a video some time ago saying that if x^x^x^x.... converges, it converges to -W(-ln(x))/ln(x) (or something like that) where W is lambert's w function.

    • @Eric4372
      @Eric4372 3 ปีที่แล้ว +5

      Yes, that’s correct
      x^x^x^x^x... = LambertW(-ln(x))/-ln(x)
      Ther answer will only be finite for x=(0, e^(1/e)] or (0, ~1.44466786]
      A power tower of e^(1/e) will converge to e. Anything greater than that will diverge to infinity.

    • @Someone-cr8cj
      @Someone-cr8cj 3 ปีที่แล้ว +3

      @@Eric4372 so cool. I love this obscure part of mathematics

    • @TomRocksMaths
      @TomRocksMaths  3 ปีที่แล้ว +1

      Agreed!

  • @redpepper74
    @redpepper74 ปีที่แล้ว

    Currently in calculus so it was cool to use it to solve this puzzle! I did
    f(x) = a^x
    *f(c)* = a^c = *c*
    a = c^(1/c)
    *f'(c)* = a^c • ln(a) = *1*
    c • ln(c^(1/c)) = 1
    ln(c) = 1
    c = e
    a = e^(1/e)

  • @wsw70
    @wsw70 2 ปีที่แล้ว

    Great video (as always from both of you). Would you know which program was used to plot the curves?

    • @TomRocksMaths
      @TomRocksMaths  2 ปีที่แล้ว +1

      Desmos - free on their website

  • @Seba_World
    @Seba_World 3 ปีที่แล้ว +1

    Tetration (towers) remainds me about how to imagine Graham's Number. Cheers from Poland. 🇵🇱🍻

  • @pinakichatterjee705
    @pinakichatterjee705 2 ปีที่แล้ว

    @Tom @3B1B I found a brilliant idea to show the lower and upper bound for a for the function a^(x) there will be two bounds naturally, the lower bound is when the curve y=x is normal to the curve a^x and the upper bound is when the curve y=x is tangent to the curve a^x. Find out the rest :3

  • @vinujavihansith1123
    @vinujavihansith1123 3 ปีที่แล้ว +5

    RIP bug.

  • @joseville
    @joseville 2 ปีที่แล้ว +1

    It wasn't explicitly stated, but the way in which function f was defined means x^x^x^x ^... means x^(x^(x^(x^(...))))
    Does that mean the ^ operator is taken to be right associative? I think it does.
    By contrast, had they defined f(x) as return x**sqrt(2), then that would mean they're treating exponentiation as left associative.

    • @ChronusZed
      @ChronusZed 2 ปีที่แล้ว +2

      I don't think the expression x^x^x^x^... really has a standard interpretation, but when you write it as a tower as in the video it's unambiguously right associative.
      Regardless, the left associative version is pretty boring. The n-th iterate just becomes x_0^(a^n), which has very easily analyzed behavior (you can end up with either 0, 1, x_0, infinity, or a simple period 2 oscillation).

  • @alexanderying1558
    @alexanderying1558 3 ปีที่แล้ว +1

    You can also ask yourself “To what power do you have to take sqrt2 to exceed 2 and how would you reach that value?” which would give you a contradiction, given you start at something smaller than 2.

    • @TomRocksMaths
      @TomRocksMaths  3 ปีที่แล้ว +1

      Be careful - it doesn't quite work like that... for example, sqrt2 to the power 8 gives you 2^4.

    • @alexanderying1558
      @alexanderying1558 3 ปีที่แล้ว +1

      @@TomRocksMaths Yeah, but to get to 8 in the first place you would need a value bigger than 2 to start with.

  • @ANunes06
    @ANunes06 3 ปีที่แล้ว +2

    Are we just not going to talk about the fact that the graph of y= sqrt(2)^x and y= x also intersect at the value of 4? Because I really wanted you to bring that home.

    • @TomRocksMaths
      @TomRocksMaths  3 ปีที่แล้ว +1

      It is a fixed point yes, BUT it is unstable so the iterative procedure will never end up there unless you start at exactly that point.

  • @kaivalya931
    @kaivalya931 ปีที่แล้ว

    when he said I didnt think about this earlier and we would have to figure it on the way
    I thought it would some scripted moment but am genuinely happy that it took longer
    this has happened to me on many occasions but thats the interesting part, the brainstorming

  • @javieracruz7172
    @javieracruz7172 3 ปีที่แล้ว +1

    Filooooo, los amo

  • @donaldhobson8873
    @donaldhobson8873 3 ปีที่แล้ว +7

    4 is a fixed point, of the iteration, just an unstable one.

  • @chrisclarkeML
    @chrisclarkeML 3 ปีที่แล้ว

    Does this hold true for all n-dimension numbers, that there is a number where the power tower does not explode to infinity or go to zero?

  • @karthikmanasali9343
    @karthikmanasali9343 10 หลายเดือนก่อน

    Name that website you guys used for graphs pls

  • @rosuav
    @rosuav 3 ปีที่แล้ว +7

    9:41 and 15:47 - 4 shows up as the other point where the graphs cross. Can you elaborate on whether it is, in some way, a valid solution?

    • @TomRocksMaths
      @TomRocksMaths  3 ปีที่แล้ว +7

      As Grant mentions at the end of the video, it is an intersection point yes, BUT it is what we call 'unstable' which means that the 'power tower' process will never converge to it unless you start on exactly that point. Initiating the sequence from any other value will see you either go to 2 (as it is a stable point) or to infinity. If you look up 'cobweb maps' you should get more of an idea about what's happening.

    • @rosuav
      @rosuav 3 ปีที่แล้ว +1

      @@TomRocksMaths Ah yep, thanks. So it's like why phi is 1.618 and not -0.618, cf th-cam.com/video/CfW845LNObM/w-d-xo.html

    • @jasonbattermann9982
      @jasonbattermann9982 3 ปีที่แล้ว

      Thanks for this question and explanation. I can see now how the region of a(x) between 2 and 4 will cobweb back to 2.

  • @UrielZyx
    @UrielZyx ปีที่แล้ว

    So if you look at the graph at 9:37, you can see why the answer of sqrt(2) popped out for the second equation.
    If you start with c_0=4, you'll be at a stable state and will continue getting 4.

    • @aprendiendoC
      @aprendiendoC ปีที่แล้ว

      Isn't that just a coincidence?

  • @khajiit92
    @khajiit92 3 ปีที่แล้ว +1

    x=4 seems to be the other intersection point of the graph, if you start your iteration at e.g. 2.5 instead of starting at 1, i think it would instead converge to 4 maybe? and then if you start iterating above 4 then it blows up. does it have to be just the first intersection?
    sqrt(2)^x = x has 2 solutions at x=2 and x=4

    • @TomRocksMaths
      @TomRocksMaths  3 ปีที่แล้ว +4

      The intersection point at x=4 is unstable when analysing it using cobweb maps, which is why that 'solution' isn't valid.

  • @trojanlol
    @trojanlol 3 ปีที่แล้ว +4

    Isn't there a mistake?
    (1) ln(a)*a^x = 1
    (2) a^x = 1/(ln(a))
    (3) x = ln(1/(ln(a)))
    If you apply ln to both sides on line (2), and since ln(a^x) = x*ln(a), shouldn't line (3) be:
    x = ln(1/(ln(a)))/ln(a)
    ???

    • @TomRocksMaths
      @TomRocksMaths  3 ปีที่แล้ว +2

      There could well be, but fortunately we didn't actually use that part in the final calculation! As we show in the video the limiting value for convergence is e^(1/e)

  • @justitroyal7032
    @justitroyal7032 3 ปีที่แล้ว +3

    If we divide one tower into two we should still get 2 for both towers so 2^2 equals 4 right

    • @TomRocksMaths
      @TomRocksMaths  3 ปีที่แล้ว +4

      Interesting idea, but I'm not sure it works quite like that... you have to be very careful when dealing with the concept of infinity, which is what Grant is hinting at when explaining the idea of defining a function that converges to 2.

    • @justitroyal7032
      @justitroyal7032 3 ปีที่แล้ว +1

      @@TomRocksMaths thanks for the reply

    • @channelnamechannel
      @channelnamechannel 3 ปีที่แล้ว +4

      i was wondering about this too, but the problem is that exponentiation isn’t associative, which means that you can’t move parentheses around without changing the result. in the video, they are talking about:
      x^(x^(x^(x^(x…
      which, for example, is not equal to:
      (x^x)^(x^x)^(x^x)…
      nor is it equal to:
      (x^x^x^x…)^(x^x^x^x…)
      which would be equal to 4 for x = √2. i thought i could be clever, so i tried rewriting the python code in the video starting with the function:
      >>> def f(x): return x**x
      but iterating this with √2 just puts larger and larger inputs into x**x. if you draw out the cobweb diagram for this, you see that it only converges for (real, non-negative) x < 1, with x = 1 being the only value for which x^x = x. all other negative/complex inputs seem to converge to 1??? (assuming you force the complex arguments to lie between -pi < Arg(z) < +pi)

    • @justitroyal7032
      @justitroyal7032 3 ปีที่แล้ว +2

      @@channelnamechannel cool

    • @siddharthberera8480
      @siddharthberera8480 3 ปีที่แล้ว +2

      @@channelnamechannel yes exactly!!!

  • @shreyanshbhandari6215
    @shreyanshbhandari6215 3 ปีที่แล้ว +1

    Was quite easy though

  • @buIlmarket
    @buIlmarket 3 ปีที่แล้ว +1

    hello nice to meet you

  • @talhabedir3812
    @talhabedir3812 2 ปีที่แล้ว

    I did not know 3blue1brown was human

  • @laviekolchinsky9441
    @laviekolchinsky9441 3 ปีที่แล้ว +3

    In terms of integer tetrations greater than 2, e.g. x^x^x, there is no general inverse like we have for the cases at 1,2, and infinity. It's transcendental. However, I've seen that it's possible to extend the inverse of the function (specifically the "super-root" inverse, instead of the logarithmic one) to these by using an extended form of the W-Lambert function. There's also no elementary integral, but I found the derivative of the integer case expressed in terms of the previous derivative. As far as I could tell and with my methods I couldn't find a neat universal form to express it, but the form I could express it in was quite simple and elegant. I'll paste it here if you're interested. As far as the integral is concerned, I currently have no idea. Do you have any knowledge on how to analyse these functions? Is there anything we can do with non-elementary/transcendental functions to generally work with them (maybe through series expressions?)? Thanks.

  • @danielrhouck
    @danielrhouck 3 ปีที่แล้ว +3

    Okay, but at 9:37 you show that graph with a = √2, and it intersects at 2 **and at 4**. Why do we say that x^x^x^x^x^… is equal to 2 in that case instead of the other possible solution; why do we have to start at 1 instead of some other place where it converges to 4?
    (I assume this has something to do with 1 being the multiplicative identity and everything, but it still seems like there should be *some* way to express that the less-preferred solution is 4 or something)

    • @OscarCunningham
      @OscarCunningham 3 ปีที่แล้ว +2

      The Taylor expansion of sqrt(2)^(4+x) begins 4 + xlog(4) + ... . Since log(4) is greater than 1 this means that if you start with a point near 4 it will move *away* from 4 when you apply the function. Whereas if you start with a point near 2 it will move towards 2. So 2 is a stable equilibrium and 4 is unstable.

    • @danielrhouck
      @danielrhouck 3 ปีที่แล้ว +2

      @@OscarCunningham Ah, that makes sesnse.

    • @TomRocksMaths
      @TomRocksMaths  3 ปีที่แล้ว +2

      What he said ^^^^^ Thank you Oscar!

    • @dr_rich_r
      @dr_rich_r 3 ปีที่แล้ว +1

      We really just need to start at sqrt(2). To see what happens to x^x^x... when x=sqrt(2), we set f(x)=sqrt(2)^x. Then f(sqrt(2)) = sqrt(2)^sqrt(2). And f(answer) = sqrt(2)^sqrt(2)^sqr(2) {somewhat ambiguous, but this problem means sqrt(2)^(sqrt(2)^sqrt(2))}. Keep taking f(answer) to check convergence. But note that starting at x=1 gives the same tower, but just one iteration behind.
      (And it's easier to change bases but start at x=1 for a general principle.)
      So there is nowhere else to start if you want to check convergence of x^x^x... at sqrt(2) using iterations of the function f(x)=sqrt(2)^x.

  • @arthurcpiazzi
    @arthurcpiazzi 2 ปีที่แล้ว

    This video changed my mind. It makes me think about the nature of infinity and the quantum nature of reality. I hope to write a paper and have you both as co-authors

    • @arthurcpiazzi
      @arthurcpiazzi 2 ปีที่แล้ว

      Either way thanks for the inspiration. Maths is beautiful

  • @ashwinkarthikeyan2355
    @ashwinkarthikeyan2355 3 ปีที่แล้ว +1

    Aren't the first 2 proofs also correct? Because the statements you proved were "If [x^(x^(x^(... ] = 4, then x = sqrt(2)" and "If [x^(x^(x^(... ] = 2, then x = sqrt(2)". Both are true statements, it's just that only one of the 2 hypotheses is true.

    • @TomRocksMaths
      @TomRocksMaths  3 ปีที่แล้ว +2

      The idea is that only one of them is possible within the setup of the problem... this is what we are alluding to when talking about cobweb diagrams and stable/unstable points.

  • @williamrutherford553
    @williamrutherford553 3 ปีที่แล้ว +1

    I don't think you ever answered why it works for 2 and not 4! The answer is, you assumed the seed was 1. Since the value wasn't e^(1/e) where it crossed at exactly one point, there's two solutions. When you come from below 2 (like 1 is) you converge upto 2. When you come from above 4, you converge down to 4.

    • @marcospasa7535
      @marcospasa7535 3 ปีที่แล้ว +1

      Actually you will diverge to infinity if you come from above 4
      For example, starting with 5:
      Step 1: (2^(1/2))^5 ≈ 5,65
      Step 2: (2^(1/2))^5,65 ≈ 7,10
      Step 3: (2^(1/2))^7,10 ≈ 11,72
      Step 4: (2^(1/2))^11,72 ≈ 58,17
      Step 5: (2^(1/2))^58,17 ≈ 82,27

    • @TomRocksMaths
      @TomRocksMaths  3 ปีที่แล้ว

      The 'solution' at 4 is an unstable point on the cobweb diagram which is why it will diverge.

  • @frankiee1550
    @frankiee1550 3 ปีที่แล้ว

    5:43 Pov: You joined the Matrix

  • @PritishMishra
    @PritishMishra 3 ปีที่แล้ว +2

    Best Colab present in TH-cam !!

  • @Pedozzi
    @Pedozzi 3 ปีที่แล้ว +2

    6:39 here i understood that this had to do with the mandelbrot set and the population equation

    • @TomRocksMaths
      @TomRocksMaths  3 ปีที่แล้ว

      Yes, that's another famous use of cobweb maps.

  • @PritishMishra
    @PritishMishra 3 ปีที่แล้ว +6

    Video Length 16:15
    Golden Ratio 🤨🤨

    • @TomRocksMaths
      @TomRocksMaths  3 ปีที่แล้ว +5

      And I thought no-one would spot it...

  • @BetaTestingUrGf
    @BetaTestingUrGf 3 ปีที่แล้ว +1

    so is it just a coincidence that the sqrt(2)^x graph intersect the y=x graph at 2 AND 4 ? 9:40

    • @TomRocksMaths
      @TomRocksMaths  3 ปีที่แล้ว +2

      4 is a solution, but it is unstable which means the iterative process that grant talks about will never end up there unless you start at exactly that point. If you start at 4 + epsilon then you never get back to 4.

    • @BetaTestingUrGf
      @BetaTestingUrGf 3 ปีที่แล้ว

      Thank you for taking the time to reply!
      Yeah i found the same question in other comments, and saw that you had replyed there :) im still not sure i understand it though 😅

  • @Frits34000
    @Frits34000 3 ปีที่แล้ว

    They didn't explain what was wronng with the 4 answer of the inteo

    • @adityakhanna113
      @adityakhanna113 3 ปีที่แล้ว +2

      They did. You can't solve an equation before knowing if the solution exists or not.

  • @gabitheancient7664
    @gabitheancient7664 2 ปีที่แล้ว

    they are math mates

  • @gurkiratsingh7tha993
    @gurkiratsingh7tha993 2 ปีที่แล้ว

    if a = e^(1/e) then a^a^a^.... = e!!!

  • @DestroManiak
    @DestroManiak 3 ปีที่แล้ว

    the felt tip pen noises kind of shredded inside my skull. great video otherwise.

  • @cjgilmore283
    @cjgilmore283 หลายเดือนก่อน

    Great. Now kiss.

  • @joseville
    @joseville 2 ปีที่แล้ว

    12:51 where does ln(a)x_0 = 1 come from??? Why gloss over the derivation?

    • @danaily.2415
      @danaily.2415 2 ปีที่แล้ว +2

      We know ln(a)*a^x_0 = 1, but a^x_0 = x_0 since the graphs y = x and y = a^x intersect at that point i.e they have the same outputs. And since the output of y = x when we input x_0 is x_0, so is the one from a^x (a^x_0 = x_0).
      In other words, at that exact point a^x outputs its input(x_0) because it intersects y = x.

  • @tyronearnold6692
    @tyronearnold6692 3 ปีที่แล้ว

    There is some terrible electronic noise throughout this

  • @rtrtrtrtrt1909
    @rtrtrtrtrt1909 3 ปีที่แล้ว

    Actually it’s not a paradox. Cause you just prove that IF x^x......^x = 4 THEN x should be equal to sqrt(2). But you still have to verify that sqrt(2) is indeed a solution. It appears that is not so this equation has no solution.

  • @anaselabid1842
    @anaselabid1842 3 ปีที่แล้ว

    At 2:32, x = Sqrt(sqrt2).

    • @TomRocksMaths
      @TomRocksMaths  3 ปีที่แล้ว +1

      I think it's still sqrt(2) as we are taking 4 to the power of 1/4, which is the same as square rotting twice.

  • @Abish_
    @Abish_ ปีที่แล้ว

    I understood nothing🤐

  • @DeadJDona
    @DeadJDona 3 ปีที่แล้ว

    💙💙💙 OTP 🤎